¿Cómo podemos demostrar que [matemáticas] n ^ 3 + 3n ^ 2 + 2n [/ matemáticas] es divisible por 3?

Bueno, echemos un vistazo. Pruebe diferentes [matemáticas] n [/ matemáticas] y vea qué sucede:

[matemáticas] n = 1: 1 + 3 + 2 = 6 [/ matemáticas]

[matemáticas] n = 2: 8 + 3 \ veces4 + 2 \ veces2 = 8 + 12 + 4 = 24 [/ matemáticas]

[matemáticas] n = 10: 1000 + 300 + 6 = 1306 [/ matemáticas]

Entonces todo eso se verifica. Por lo tanto, podría valer la pena pensar en una prueba.

[matemática] n ^ 3 + 3n ^ 2 + 2n [/ matemática] también se puede escribir como [matemática] n (n ^ 2 + 3n + 2) [/ matemática] y eso se puede factorizar en

[matemáticas] n (n ^ 2 + 3n + 2) = n (n + 1) (n + 2) [/ matemáticas]

El lado derecho son tres enteros consecutivos, por lo que uno de ellos es divisible por tres, por lo que cualquier producto que tenga ese número como factor es divisible por tres. Hecho.

Por supuesto, hay muchas maneras de desollar a ese gato en particular, así que intentemos la inducción matemática. Necesitamos anclar la inducción con un comienzo; afortunadamente, ya sabemos que para [matemáticas] n = 1, f (n) = n ^ 3 + 3n ^ 2 + 2n [/ matemáticas] es igual a 6 y, por lo tanto, es divisible por tres:

[matemáticas] f (1) = 6 [/ matemáticas]

Ahora supongamos que [math] f (n) [/ math] es divisible por tres y demuestre que en ese caso [math] f (n + 1) [/ math] también es divisible por tres:

[matemáticas] \ begin {align}
f (n + 1) & = (n + 1) ^ 3 + 3 (n + 1) ^ 2 + 2 (n + 1) \\
& = n ^ 3 + 3n ^ 2 + 3n + 1 + 3 (n ^ 2 + 2n + 1) + 2n + 2 \\
& = n ^ 3 + 3n ^ 2 + 2n + (3n ^ 2 + 9n + 3) \\
& = f (n) + 3 (n ^ 2 + 3n + 1)
\ end {align} [/ math]

Como [matemática] f (n) [/ matemática] es divisible por tres y [matemática] 3 (n ^ 2 + 3n + 1) [/ matemática] es explícitamente divisible por tres, también lo es [matemática] f (n + 1 )[/matemáticas]. Hecho.

Teorema: Exactamente uno de los [números matemáticos] n [/ matemáticas] consecutivos es divisible por [matemáticas] n [/ matemáticas]

[matemáticas] \ begin {align} n ^ 3 + 3n ^ 2 + 2n & = n (n ^ 2 + 3n + 2) \\ & = n (n + 1) (n + 2) \ end {align} \ tag *{}[/matemáticas]

[matemática] n, n + 1 [/ matemática] y [matemática] n + 2 [/ matemática] son ​​[matemática] 3 [/ matemática] enteros consecutivos. Según el teorema, exactamente uno de estos es divisible por [matemáticas] 3 [/ matemáticas]. Para probar esto más rigurosamente, elegimos [matemática] n = \ left \ {3k-1,3k, 3k + 1 \ right \}, k \ in \ Z [/ math] y sustituimos. Cada vez, tendremos un número que es múltiplo de [math] 3 [/ math].

Esto termina la prueba.

Sí, podemos demostrar que [matemáticas] n ^ 3 + 3n ^ 2 + 2n [/ matemáticas] es divisible por 3.

Tenemos:

[matemáticas] n ^ 3 + 3n ^ 2 + 2 = n (n ^ 2 + 3n + 2) = n (n ^ 2 + 2n + n + 2) = n (n (n + 2) + 1 (n + 2)) = n (n + 1) (n + 2) [/ matemáticas]

Entonces, la expresión dada es producto de tres números consecutivos. El producto de 3 números consecutivos siempre es un múltiplo de 3 (también es cierto para otros números). Déjame probar esto también:

Podemos escribir cualquier número como [math] 3k, (3k + 1), (3k + 2) [/ math]. Vamos a sustituir uno por uno, cada una de estas 3 posibilidades en nuestra expresión original.

  • n = 3k
    En este caso nuestra expresión se convierte en:
    [matemáticas] 3k (3k + 1) (3k + 2) [/ matemáticas]
    que es claramente divisible por 3.
  • n = (3k + 1)
    En este caso nuestra expresión se convierte en:
    [matemáticas] (3k + 1) ((3k + 1) +1) ((3k + 1) +2) = (3k + 1) (3k + 2) (3k + 3) = (3k + 1) (3k +2) .3 (k + 1) = 3 (k + 1) (3k + 1) (3k + 2) [/ matemáticas]
    que también es claramente divisible por 3.
  • n = (3k + 2)
    En este caso nuestra expresión se convierte en:
    [matemáticas] (3k + 2) ((3k + 2) +1) ((3k + 2) +2) = (3k + 2) (3k + 3) (3k + 4) = (3k + 2) .3 (k + 1) (3k + 4) = 3 (k + 1) (3k + 2) (3k + 4) [/ matemáticas]
    que también es claramente divisible por 3.

Por lo tanto demostrado, [matemática] n ^ 3 + 3n ^ 2 + 2n [/ matemática] es divisible por 3.

Supongamos que el múltiplo de 3 se escribe como [math] M_3 [/ math] y:

[matemáticas] X = n ^ 3 + 3 n ^ 2 + 2n [/ matemáticas]

El segundo término ya es un múltiplo de 3, por lo que puede escribirse como:

[matemáticas] n ^ 3 + 2n + M_3 [/ matemáticas]

[matemáticas] = n (n ^ 2 + 2) + M_3 [/ matemáticas]

Cualquier múltiplo de 3 se puede definir como [matemática] n ^ 2 + 2 [/ matemática] donde, [matemática] n \ en N [/ matemática] y [matemática] n \ neq M_3 [/ matemática]

Por lo tanto,

[matemáticas] X = n \ veces M_3 + M_3 [/ matemáticas]

Si n no es un múltiplo de 3, y,

[matemáticas] X = M_3 \ veces (n ^ 2 + 2) + M_3 [/ matemáticas]

Si n es un múltiplo de 3.

Así,

[matemáticas] X = M_3 + M_3 [/ matemáticas]

Por lo tanto,

X es un múltiplo de 3, o

[matemáticas] \ en caja {X = M_3} [/ matemáticas]

Nota: Esta prueba se divide en diferentes partes dependiendo de qué tan profunda sea la declaración que está tratando de probar. Si solo lo desea para [math] n \ in \ mathbb {Z} [/ math], mire el primero. Si desea [math] n = a + bi \ in \ mathbb {C} [/ math] y [math] a, b \ in \ mathbb {Z} [/ math], mire el segundo. Si desea [math] n = a + bi \ in \ mathbb {C} [/ math] con [math] a, b \ in \ mathbb {QZ} [/ math], mire el tercero. Si desea [math] n = a + bi \ in \ mathbb {C} [/ math] con [math] a, b \ in \ mathbb {RQ} [/ math], mire el cuarto.

Además, solo las primeras pruebas [matemáticas] 2 [/ matemáticas] se han completado hasta ahora.

Agregue un voto a favor por las [matemáticas] 3 [/ matemáticas] horas que pasé escribiendo todo esto en látex, etc. 🙂


Introducción general

Definir [matemáticas] f (n) = n ^ 3 + 3n ^ 2 + 2n = n (n ^ 2 + 3n + 2) = n (n + 1) (n + 2) [/ matemáticas]


Prueba 1

Primero hagamos [math] f (n) [/ math] para [math] n \ in \ mathbb {Z} [/ math].

Usando inducción:

[matemáticas] f (1) = 1 + 3 + 2 = 6 [/ matemáticas], que es divisible por [matemáticas] 3 [/ matemáticas].

Suponiendo que [math] f (n) [/ math] funciona para todos los enteros menores que [math] k [/ math]:

[matemáticas] f (k + 1) = (k + 1) \ left ((k + 1) +1 \ right) \ left ((k + 1) +2 \ right) = (k + 1) (k + 2) (k + 3) = k (k + 1) (k + 2) + 3 (k + 1) (k + 2) [/ matemáticas].

Como [matemática] k (k + 1) (k + 2) = f (k) [/ matemática] es divisible por [matemática] 3 [/ matemática] por la hipótesis de inducción, [matemática] f (k + 1) [ / math] es divisible por [math] 3 [/ math].

Por lo tanto, por inducción, [matemática] f (n) [/ matemática] es divisible por [matemática] 3 [/ matemática] para todos los enteros positivos [matemática] n [/ matemática].

Ahora suponga que [math] f (n) [/ math] funciona para todos los enteros mayores que [math] k [/ math]:

[matemáticas] f (k-1) = (k-1) \ left ((k-1) +1 \ right) \ left ((k-1) +2 \ right) = (k + 2-3) ( k) (k + 1) = k (k + 1) (k + 2) – 3k (k + 1) [/ matemáticas].

Como [matemática] k (k + 1) (k + 2) = f (k) [/ matemática] es divisible por [matemática] 3 [/ matemática] por la hipótesis de inducción, [matemática] f (k-1) [ / math] es divisible por [math] 3 [/ math].

Por lo tanto, por inducción, [matemática] f (n) [/ matemática] es divisible por [matemática] 3 [/ matemática] para todos los enteros negativos [matemática] n [/ matemática].

Por lo tanto, combinando ambas inducciones, [math] f (n) [/ math] es verdadero para [math] n \ in \ mathbb {Z} [/ math].


Introducción para la Prueba 2 + 3 + 4

Ahora para demostrar que es o no integral para no integrales [math] n [/ math]:

Tome [math] real (x) [/ math] como la parte real de [math] x [/ math] y [math] imag (x) [/ math] la parte imaginaria de [math] x [/ math] .

Tome [math] real (x) = a [/ math] y [math] imag (x) = b [/ math] para [math] a, b \ in \ mathbb {R} [/ math].

Entonces, [matemáticas] n = a + bi [/ matemáticas].


Prueba 2

Primero hagamos esto para [math] a, b \ in \ mathbb {Z} \ space ([/ math] Podemos porque [math] \ mathbb {Z} \ subset \ mathbb {R} [/ math] y lo haremos porque entonces esta parte de la prueba es más fácil [matemáticas]) [/ matemáticas].

[matemáticas] f (n) = \ left (a + bi \ right) \ left ((a + 1) + bi \ right) \ left ((a + 2) + bi \ right) [/ math].

Por lo tanto, [matemáticas] f (n) = \ left (a + bi \ right) \ left (a + 1 + bi \ right) \ left (a + 2 + bi \ right) = a ^ 3 + 3a ^ 2bi + 3a ^ 2 + 3ab ^ 2i ^ 2 + 6abi + 2a + b ^ 3i ^ 3 + 3b ^ 2i ^ 2 + 2bi = a ^ 3 + 3a ^ 2-3ab ^ 2 + 2a-3b ^ 2 + 3a ^ 2bi + 6abi-b ^ 3i + 2bi = \ left (a ^ 3 + 3a ^ 2-3ab ^ 2 + 2a-3b ^ 2 \ right) + b \ left (3a ^ 2 + 6a-b ^ 2 + 2 \ right) i \ space [/ math]. [math] * [/ math]

Al observar la parte imaginaria de [matemáticas] f (n) [/ matemáticas] primero, vemos que es [matemáticas] b \ izquierda (3 (a ^ 2 + 2a) -b ^ 2 + 2 \ derecha) [/ matemáticas].

Tiempo para el trabajo de casos en [matemáticas] b [/ matemáticas] porque [matemáticas] b [/ matemáticas] es la única variable que afecta la parte imaginaria:

  • [matemáticas] b \ equiv 0 \ espacio (mod \ espacio 3) [/ matemáticas]:
  • Como toda la parte imaginaria tiene un factor de [matemática] b [/ matemática], [matemática] imag \ left (f (n) \ right) \ equiv 0 \ space (mod \ space 3) [/ math].
  • Es divisible por [matemáticas] 3 [/ matemáticas].
  • [matemáticas] b \ equiv 1 \ espacio (mod \ espacio 3) [/ matemáticas]:
    • [matemáticas] imag \ left (f (n) \ right) = 3 (a ^ 2b + 2ab) -b ^ 3 + 2b \ equiv 2b-b ^ 3 \ equiv 2 \ times 1-1 ^ 3 \ space \ space \ space \ space \ equiv 2-1 \ equiv 1 \ space (mod \ space 3) [/ math].
    • No es divisible por [matemáticas] 3 [/ matemáticas].
  • [matemáticas] b \ equiv 2 \ espacio (mod \ espacio 3) [/ matemáticas]:
    • [matemáticas] imag \ left (f (n) \ right) = 3 (a ^ 2b + 2ab) -b ^ 3 + 2b \ equiv 2b-b ^ 3 \ equiv 2 \ times 2-2 ^ 3 \ space \ space \ space \ space \ equiv 4-8 \ equiv 2 \ space (mod \ space 3) [/ math].
    • No es divisible por [matemáticas] 3 [/ matemáticas].

    Ahora en la parte real de [matemáticas] f (n) [/ matemáticas]:

    [matemática] real \ izquierda (f (n) \ derecha) = a ^ 3 + 3a ^ 2-3ab ^ 2 + 2a-3b ^ 2 = a ^ 3 + 2a + 3 (a ^ 2-ab ^ 2-b ^ 2) = a \ left (a ^ 2 + 2 \ right) +3 \ left (a ^ 2-ab ^ 2-b ^ 2 \ right) [/ math].

    [matemática] real \ left (f (n) \ right) = a \ left (a ^ 2 + 2 \ right) +3 \ left (a ^ 2-ab ^ 2-b ^ 2 \ right) \ equiv a \ left (a ^ 2 + 2 \ right) \ space (mod \ space 3) [/ math].

    Tiempo para el trabajo de casos en [matemáticas] a [/ matemáticas] porque [matemáticas] a [/ matemáticas] es la única variable que afecta la parte real:

    • [matemáticas] a \ equiv 0 \ espacio (mod \ espacio 3) [/ matemáticas]:
    • Como toda la parte real módulo [matemáticas] 3 [/ matemáticas] tiene un factor de [matemáticas] a [/ matemáticas], [matemáticas] real \ izquierda (f (n) \ derecha) \ equiv 0 \ espacio (mod \ espacio 3) [/ matemáticas].
    • Es divisible por [matemáticas] 3 [/ matemáticas].
  • [matemáticas] a \ equiv 1 \ espacio (mod \ espacio 3) [/ matemáticas]:
    • [matemáticas] real \ izquierda (f (n) \ derecha) \ equiv a ^ 3 + 2a \ equiv 1 ^ 3 + 2 \ veces 1 \ equiv 1 + 2 \ equiv 0 \ space (mod \ space 3) [/ math ]
    • Es divisible por [matemáticas] 3. [/ Matemáticas]
  • [matemáticas] a \ equiv 2 \ espacio (mod \ espacio 3) [/ matemáticas]:
    • [matemáticas] real \ izquierda (f (n) \ derecha) \ equiv a ^ 3 + 2a \ equiv 2 ^ 3 + 2 \ veces 2 \ equiv 8 + 4 \ equiv 0 \ space (mod \ space 3) [/ math ]
    • Es divisible por [matemáticas] 3 [/ matemáticas].

    Por lo tanto, para [math] a, b \ in \ mathbb {Z} [/ math], [math] f (n) = f \ left (a + bi \ right) \ equiv 0 \ space (mod \ space 3) [/ math] solo si [math] b \ equiv 0 \ space (mod \ space 3) [/ math].


    Prueba 3 (continuará …)

    Ahora para [math] n = a + bi [/ math] donde [math] a, b \ in \ mathbb {QZ} [/ math].


    Prueba 4 (continuará …)

    Ahora para [math] n = a + bi [/ math] donde [math] a, b \ in \ mathbb {RQ} [/ math] [math]. [/ Math]


    [matemáticas] * [/ matemáticas] Por favor revise mis matemáticas aquí 🙂

    ¿Cómo podemos demostrar que [matemáticas] n ^ 3 + 3n ^ 2 + 2n [/ matemáticas] es divisible por 3?

    Un par de excelentes respuestas hasta ahora usando inducción y factorización, pero nadie ha mencionado aún el pequeño teorema de Fermat.

    Según Fermat, [matemáticas] n ^ 3-n [/ matemáticas] es divisible por 3, ya que 3 es primo. Así es [matemáticas] 3n ^ 2 + 3n [/ matemáticas], obviamente. Agregar estos dos múltiplos de 3 da otro múltiplo de 3, que es la expresión que estamos buscando.

    EDITAR: Se me ocurre que hay una manera más fácil de usar el mismo teorema:

    [matemáticas] n ^ 3–3n ^ 2 + 2n = (n + 1) ^ 3- (n + 1) [/ matemáticas]

    Su problema se reduce a mostrar que [matemáticas] n ^ 3 + 2n = n (n ^ 2 + 2) [/ matemáticas] es divisible por [matemáticas] 3 [/ matemáticas].

    Si [math] n [/ math] es un múltiplo de 3, entonces hemos terminado. Si no, puede mostrarlo manualmente o invocar el pequeño teorema de Fermat para concluir que [math] n ^ 2 \ equiv 1 \ pmod {3} [/ math]. Por lo tanto, [math] n ^ 2 + 2 \ equiv 0 \ pmod {3} [/ math] que significa 3 divide [math] n ^ 2 + 2 [/ math].

    Por lo tanto, hemos demostrado que 3 divide [matemáticas] n (n ^ 2 + 2) [/ matemáticas] para cualquier número entero n. Así sigue la conclusión que deseabas.

    Factor primero

    [matemáticas] n ^ 3 + 3n ^ 2 + 2n = [/ matemáticas]

    [matemáticas] n (n ^ 2 + 3n +2) = [/ matemáticas]

    [matemáticas] n (n + 1) (n + 2) [/ matemáticas] Cualquier conjunto de tres enteros consecutivos incluirá exactamente un múltiplo de tres. Cada tercer entero es un múltiplo de tres. Entonces, dado que estamos multiplicando tres enteros consecutivos aquí, sabemos exactamente que uno de ellos es un múltiplo de tres y, por lo tanto, el producto de los números consecutivos contiene al menos un factor de 3, lo que también lo convierte en un múltiplo de 3.

    Por inducción matemática. Eso es resolver la ecuación sustituyendo n por 1,2,3 … k & k + 1 en ese orden y verificando si cada respuesta es divisible por 3. Si se satisface usando esos valores, implica que la ecuación es divisible por 3